Đến nội dung

Hình ảnh

M=$\frac{3a^{4}+3b^{4}+c^{3}+2}{(a+b+c)^{3}}$


  • Please log in to reply
Chủ đề này có 4 trả lời

#1
hien2000a

hien2000a

    Thượng sĩ

  • Thành viên
  • 234 Bài viết

Cho 3 số thực dương a,b,c. Tìm giá trị nhỏ nhất của biểu thức :

M=$\frac{3a^{4}+3b^{4}+c^{3}+2}{(a+b+c)^{3}}$


~O)  ~O)  ~O)  CHÚNG TA KHÔNG THỂ THAY ĐỔI QUÁ KHỨ NHƯNG CÓ THỂ THAY ĐỔI CẢ TƯƠNG LAI :like  :like  :like 


#2
githenhi512

githenhi512

    Thượng sĩ

  • Thành viên
  • 290 Bài viết

Cho 3 số thực dương a,b,c. Tìm giá trị nhỏ nhất của biểu thức :

M=$\frac{3a^{4}+3b^{4}+c^{3}+2}{(a+b+c)^{3}}$

Là $c^3$ hay $25c^3$ vậy. Nếu là $25c^3$ thì ở đây rồi


'' Ai cũng là thiên tài. Nhưng nếu bạn đánh giá một con cá qua khả năng trèo cây của nó, nó sẽ sống cả đời mà tin rằng mình thực sự thấp kém''.

Albert Einstein                               


#3
the unknown

the unknown

    Thượng sĩ

  • Thành viên
  • 208 Bài viết

Cho 3 số thực dương a,b,c. Tìm giá trị nhỏ nhất của biểu thức :

M=$\frac{3a^{4}+3b^{4}+c^{3}+2}{(a+b+c)^{3}}$

Sử dụng bất đẳng thức AM-GM cho bộ $4$ số ta được: $a^4+a^4+a^4+1\geq 4\sqrt[4]{a^{12}}=4a^3$. Và tương tự ta cũng được $3b^4+1\geq 4b^3$. Do đó: $3a^4+3b^4+c^3+2\geq 4a^3+4b^3+c^3$.

Đến đây sử dụng BĐT Holder ta được: $(4a^3+4b^3+c^3)(1+1+2)(1+1+2)\geq 4(a+b+c)^3\Rightarrow \frac{4a^3+4b^3+c^3}{(a+b+c)^3}\geq \frac{1}{4}$.

Đẳng thức xảy ra khi $a=b=1,c=2$.

Vậy......


$\texttt{If you don't know where you are going, any road will get you there}$


#4
hien2000a

hien2000a

    Thượng sĩ

  • Thành viên
  • 234 Bài viết

Là $c^3$ hay $25c^3$ vậy. Nếu là $25c^3$ thì ở đây rồi

c3


~O)  ~O)  ~O)  CHÚNG TA KHÔNG THỂ THAY ĐỔI QUÁ KHỨ NHƯNG CÓ THỂ THAY ĐỔI CẢ TƯƠNG LAI :like  :like  :like 


#5
hien2000a

hien2000a

    Thượng sĩ

  • Thành viên
  • 234 Bài viết

Sử dụng bất đẳng thức AM-GM cho bộ $4$ số ta được: $a^4+a^4+a^4+1\geq 4\sqrt[4]{a^{12}}=4a^3$. Và tương tự ta cũng được $3b^4+1\geq 4b^3$. Do đó: $3a^4+3b^4+c^3+2\geq 4a^3+4b^3+c^3$.

Đến đây sử dụng BĐT Holder ta được: $(4a^3+4b^3+c^3)(1+1+2)(1+1+2)\geq 4(a+b+c)^3\Rightarrow \frac{4a^3+4b^3+c^3}{(a+b+c)^3}\geq \frac{1}{4}$.

Đẳng thức xảy ra khi $a=b=1,c=2$.

Vậy......

bạn có thể viết tông quát dạng BDT Holder bạn dùng không


~O)  ~O)  ~O)  CHÚNG TA KHÔNG THỂ THAY ĐỔI QUÁ KHỨ NHƯNG CÓ THỂ THAY ĐỔI CẢ TƯƠNG LAI :like  :like  :like 





0 người đang xem chủ đề

0 thành viên, 0 khách, 0 thành viên ẩn danh